0
$\begingroup$

I'm having a hard time understanding the solution to the following problem from Dan Stefanica's book "A Primer for the Mathematics of Financial Engineering":

Call options with strikes 100, 120, and 130 on the same underlying asset and with the same maturity are trading for 8, 5, and 3, respectively (there is no bid-ask spread). Is there an arbitrage opportunity present? If yes, how can you make a riskless profit?

To model the situation, we assume $x_1, x_2, x_3$ units of the three options are purchased (where the $x_i$ may be negative as well). Then, the cost of the portfolio is $8x_1 + 5x_2 + 3x_3$, which we want to be negative for an arbitrage opportunity to exist. So we get our first equation

$$8x_1 + 5x_2 + 3x_3 < 0$$

Then we model the payoff of the portfolio at maturity as $$C(T) = x_1 max(S(T) - K_1, 0) + x_2 max(S(T) - K_2, 0) + x_3 max(S(T) - K_3, 0)$$. This gives us a piecewise description of the payoff -

\begin{equation} C(T) = \left\{\begin{array}{lr} 0, & S(T) \le K_1 \\ x_1(S(T) - K_1), & K_1 \le S(T) \le K_2\\ (x_1 + x_2)S(T) - x_1K_1 - x_2K_2, & K_2 \le S(T) \le K_3\\ (x_1 + x_2 + x_3)S(T) - x_1K_1 - x_2K_2 - x_3K_3, & K_3 \le S(T) \end{array}\right. \end{equation}

Each of these should be non-negative if an arbitrage opportunity is to exist. If we set $S(T) = K_3$, then we get $(x_1 + x_2)K_3 - x_1K_1 - x_2K_2 \geq 0$. Substituting the actual numbers in, we get $30x_1 + 10x_2 \geq 0$.

Now, here comes my confusion - the solution states that from the payoff description, we can also come up with a third condition $x_1 + x_2 + x_3 \geq 0$. I'm not sure how to arrive at this condition based on the current information we have. Any thoughts?

$\endgroup$
4
  • $\begingroup$ Why don't you try with brute force and ask the Excel solver to find the values $x_1, x_2, x_3$ to create an arbitrage? Add a few constraints to ensure the payoff is positive for values like 90, 91, 92, .. 138, 139, 140. $\endgroup$ Commented Apr 25 at 6:48
  • $\begingroup$ You need each payoff to be positive in each area of the terminal payoff function. So the first condition implies that $x_1>0$, the next condition implies that $x_1+x_2>0$ etc $\endgroup$ Commented Apr 25 at 8:05
  • 1
    $\begingroup$ Look at your last equation: if $x_1+x_2+x_3<0$, then for sufficiently high $S(T)$ the payoff will be negative. $\endgroup$ Commented Apr 26 at 12:52
  • $\begingroup$ Hi @dm63, Thank you! I do see that for sufficiently high S(T) the third expression will become negative. This helps a lot. $\endgroup$ Commented Apr 29 at 5:34

0

Your Answer

By clicking “Post Your Answer”, you agree to our terms of service and acknowledge you have read our privacy policy.

Start asking to get answers

Find the answer to your question by asking.

Ask question

Explore related questions

See similar questions with these tags.